cyruswhittaker
Thanks Received: 107
Forum Guests
 
Posts: 246
Joined: August 11th, 2010
 
 
trophy
Most Thanked
trophy
First Responder
 

Q23 - The workers at Bell Manufacturing

by cyruswhittaker Thu Sep 02, 2010 11:33 pm

I chose D after running out of time, but I see that it really is not a sufficient assumption, and not really even a necessary assumption as the argument doesn't explicitly state that the president is the one who is offering the wage increase.

However, just wanted to clarify my reasoning for C:

The first sentence introduces "unless..." indicating that it is the necessary condition, and not going on strike is the sufficient condition. So C basically states that this sufficient condition occurs, ensuring the necessary condition to occur (wage increases), and the rest of the argument follows.

Is this reasoning correct? I'm trying to get more exact on the necessary/sufficient conditions in arguments.
User avatar
 
bbirdwell
Thanks Received: 864
Atticus Finch
Atticus Finch
 
Posts: 803
Joined: April 16th, 2009
 
This post thanked 2 times.
 
 

Re: Q23 - The workers at Bell Manufacturing

by bbirdwell Sat Sep 04, 2010 12:42 pm

Let's look at the argument.

Here's the first statement and its contrapositive:
~increase wages --> strike
~strike --> increase wages

We also know:
increase wages --> sell off subsidiaries

Conclusion:
sell of subsidiaries

Now, what information must we have in order for our chain of evidence to definitively lead to this conclusion?
Well, if the company is going to increase wages, then the conclusion is drawn.
Also, if there is no strike, the conclusion will be drawn.

That's (C).
I host free online workshop/Q&A sessions called Zen and the Art of LSAT. You can find upcoming dates here: http://www.manhattanlsat.com/zen-and-the-art.cfm
 
lhermary
Thanks Received: 10
Atticus Finch
Atticus Finch
 
Posts: 160
Joined: April 09th, 2011
 
 
 

Re: Q23 - The workers at Bell Manufacturing

by lhermary Thu May 09, 2013 4:38 pm

Why is the answer not E?

I drew inferences and arrived at C but I cannot figure out why E is wrong.
Is it because it is not sufficient?
 
sumukh09
Thanks Received: 139
Atticus Finch
Atticus Finch
 
Posts: 327
Joined: June 03rd, 2012
 
 
trophy
Most Thanked
trophy
First Responder
 

Re: Q23 - The workers at Bell Manufacturing

by sumukh09 Thu May 09, 2013 8:43 pm

Won't Strike ---> Wage Increase

Wage Increase ---> Sell Subsidiaries

Therefore:

Sell Subsidiaries

Given the chain above, you have to assume that they won't strike in order to arrive at "sell subsidiaries."

E is out of scope, "accepting a package of improved benefits in place of their wage increase" is not mentioned once the argument. It doesn't help us get to "sell subsidiaries" in any way.
User avatar
 
ohthatpatrick
Thanks Received: 3808
Atticus Finch
Atticus Finch
 
Posts: 4661
Joined: April 01st, 2011
 
This post thanked 1 time.
 
 

Re: Q23 - The workers at Bell Manufacturing

by ohthatpatrick Fri May 10, 2013 12:04 am

Yeah, (D) and (E) are answer choices that sound like Necessary Assumption and/or Strengthen.

This highlights how critical it is to understand the task at hand, because if we just read for ideas that 'click', we'll pick answer choices that might be right on a different question type.

(E) sounds like it's ruling out a potential alternative, which is often how Strengthen and Necessary Assumption function.

Consider this fake argument:
Bob is crying. Thus, he must have just broken up with his girlfriend.

(A) Bob was not just cutting onions.

This would be correct as a Necessary Assumption / Strengthen answer, but this in no way PROVES that Bob broke up with his girlfriend.

As the previous poster mentioned, we have no need for any new ideas when we're doing Sufficient Assumption. We only need to connect the dots of the ideas they gave us.

We have to add the answer choice into the argument and make it so that the conclusion is 100% derived.

The premises gave us a chain that said
~Strike --> wage increases --> sell subsidiaries.

The conclusion is trying to prove "sell subsidiaries".

There are only 2 possible answers, given what they gave us.

They either tell us "there will be wage increases" or "there will not be a strike".

If you're accustomed to working from wrong to right, you might want to try changing that thinking for Sufficient Assumption.

Because our task is so specific for Sufficient Assumption, we can actually "solve for our answer" before we look at the answer choices. There are no shades of grey with the other answer choices. All the wrong answers fail to do the job. They DON'T prove the conclusion.

If I said
Prem: 7
+
Prem: 5
=
Conc: 16

and asked "what extra idea would prove the conclusion?" You wouldn't just go to the answer choices; you would first solve for the missing idea ... 7 + 5 = 12, so I'm missing +4 to get to my conclusion. Then you would just go find that answer.

I would recommend you try doing a dozen or so Sufficient Assumption questions and approach it with this very proactive sense of specifically predicting your answer before you look at answer choices. It will help you to clarify the task involved in these questions.

Let me know if you have any questions about that.
User avatar
 
WaltGrace1983
Thanks Received: 207
Atticus Finch
Atticus Finch
 
Posts: 837
Joined: March 30th, 2013
 
 
trophy
Most Thanked
trophy
Most Thankful
trophy
First Responder
 

Re: Q23 - The workers at Bell Manufacturing

by WaltGrace1983 Thu Jan 16, 2014 3:30 pm

The conditional logic here looks like this:

~Increase in wages --> Strike

or

~Strike --> Increase in wages


Then we get another tidbit of information as we read further.

Increase wages --> Sell off some subsidiaries

or

~Sell off some subsidiaries --> ~Increase wages


Let's compress this down.

___________________________________________

~Sell --> ~Increase --> Strike

and

~Strike --> Increase in wages --> Sell


We can get at this because we were able to link up the conditional reasoning provided. Therefore, we know that the link that we need to find is an answer choice linking the wages/strike to selling. We know that if the wages are increased OR there is no strike the subsidiaries must be sold. The argument is basically assuming that there is a reason why these subsidiaries are sold. This reason could be that there is no strike or that there was an increase in wages.

(A) Increased losses? What is this from?
(B) This is saying "~Increase." Well all we know about this is that if ~I then there will be a strike. This doesn't link us up to that subsidiaries part.
(D) This could be a necessary assumption if the argument talked about how only the president can decrease wages or something. Either way, this is not sufficient. It doesn't link anything up to anything.
(E) Just gives us an extra reason to believe why the subsidiaries will be sold but who is to say that these employees will not accept improved benefits but WILL accept a rubber chicken? If having a rubber chicken had the same effect as an increased wage then who is to say that the subsidiaries will be sold?

(C) Yes! This gives us ~Strike. We know....~Strike --> Increase in wages --> Sell